LSAT and Law School Admissions Forum

Get expert LSAT preparation and law school admissions advice from PowerScore Test Preparation.

User avatar
 Dave Killoran
PowerScore Staff
  • PowerScore Staff
  • Posts: 5852
  • Joined: Mar 25, 2011
|
#42634
martinbeslu wrote:My assumption was that a talented manager would grow the revenue and profitability, not just maintain it.

Similarly, if the stock price of a company stayed at $50/share for a decade everyone would sell the stock. And that is exactly what would happen if the revenue and profitability never increased. The share price must go up over time (or at least be expected to go up over time) in order for the company to stay in existence.

Answer choice D has no more affect on the argument than saying that the company won’t go bankrupt if the company’s best managers are assigned elsewhere. That’s great… but it doesn’t tell us that the less talented managers won’t do a much worse job with the established high-revenue product lines than the talented managers.
Hi Martin,

A few thoughts here. First, I have to disagree with your assessment of (D), in part because you are reading quite a bit into the circumstances here. I don't mean that in an argumentative manner, but rather a reflective manner because I personally have no stake in which answer is correct or incorrect, but I'm seeing you argue with the answer here as opposed to figuring out why the test makers say it is correct. I've written about this mindset before, and it's worth a read if you haven't seen it: You Can't Argue With the LSAT.

Second, if we go into the language if (D) itself, the idea of maintaining current revenue and profitability isn't antithetical with what you stated about growth (by hard number definition, growing something establishes that you also met the prior numbers), but regardless, growth isn't always expected or an absolute requirement as you suggest. You made a universal assumption that "a talented manager would grow the revenue and profitability" as if it was a constant certainty, but it may be that the most talented manager in the world is required simply to maintain the status quo (changing markets, disruptive forces like new technology and competitors, etc all are very real possibilities that could make "growth" nearly impossible).

When we look at the stimulus, you can see that the author has overlooked that idea that the reason the talented managers are being assigned to the "established high-revenue product lines" is to help maintain those lines (and thus avoid financial damage to the overall bottom line). The author more or less says, "Stop assigning these high-talent managers to the established and high revenue lines, and instead assign them to the new ventures where skill is critical." Well, one weakness of that argument is that if those managers aren't assigned to those top lines, there might be a decline in those lines. This is the weakness that (D) is attempting to shore up: if you assign those managers elsewhere, you won't see a degradation in those lines. I don't see that as ineffective at all, but rather central to maintaining the integrity of the argument in the stimulus.

I think the key here is that you wanted stronger language in (D), but using the point in the prior paragraph, it's actually ok that the language is lessened as it is in answer choice (D). Regardless, the key is always to try to determine why they think the answer is right. The more you do that and get into their mindset, the more success you will have.

Thanks!

Get the most out of your LSAT Prep Plus subscription.

Analyze and track your performance with our Testing and Analytics Package.